Find Corresponding Eigenvectors for Matrices A and B | Quick Help

  • Thread starter Thread starter rock.freak667
  • Start date Start date
  • Tags Tags
    Eigenvector
Click For Summary
The discussion focuses on finding eigenvectors for matrices A and B, where A has eigenvalues 1, 2, and 4. Corresponding eigenvectors for A are provided, and it is noted that B can be expressed as A + 3I, leading to a straightforward method for determining B's eigenvalues as 4, 5, and 7. The relationship between the eigenvectors of A and B is clarified, indicating that the eigenvectors remain unchanged while the eigenvalues shift due to the addition of 3. The conversation emphasizes using the definitions of eigenvalues and eigenvectors rather than relying solely on determinant calculations. This approach simplifies the process of finding eigenvalues and confirms that the eigenvectors for both matrices are the same.
rock.freak667
Homework Helper
Messages
6,221
Reaction score
31

Homework Statement



The matrix,A,given by
<br /> A = \left(<br /> \begin{array}{ccc}<br /> 7 &amp; -4 &amp; 6\\<br /> 2 &amp; 2 &amp; 2 \\<br /> -3 &amp; 4 &amp; -2 \<br /> \end{array}<br /> \right)<br />

has eigenvalues 1,2,4 . Find a set of corresponding eigenvectors.

Hence find the eigenvalues of B, where

<br /> B = \left(<br /> \begin{array}{ccc}<br /> 10 &amp; -4 &amp; 6\\<br /> 2 &amp; 5 &amp; 2 \\<br /> -3 &amp; 4 &amp; 1 \<br /> \end{array}<br /> \right)<br />

and state a corresponding set of eigenvectors.

Homework Equations





The Attempt at a Solution




Well I easily found the eigenvectors
<br /> \lambda=1 corresponds to
<br /> \left(<br /> \begin{array}{c}<br /> -1\\<br /> 0 \\<br /> 1\<br /> \end{array}<br /> \right)<br />

<br /> \lambda=2 corresponds to
<br /> \left(<br /> \begin{array}{c}<br /> -4\\<br /> 1 \\<br /> 4\<br /> \end{array}<br /> \right)<br />

<br /> \lambda=4 corresponds to
<br /> \left(<br /> \begin{array}{c}<br /> 2\\<br /> 3 \\<br /> 1\<br /> \end{array}<br /> \right)<br />


Well for the one with B, just solve det(b-\lambdaI)=0 to get the e.values... but it says to state a set of e.vectors meaning that I am not supposed to work them out.
The only thing I can really say about A and B is that in B all the elements in the main diagonal are the elements in the main diagonal of A with 3 added to them
 
Physics news on Phys.org
In other words, B = A + 3I. Can you find a simple way to get B's eigenvalues and eigenvectors without explicitly working through B? Use the definitions of eigenvectors and eigenvalues.
 
slider142 said:
In other words, B = A + 3I. Can you find a simple way to get B's eigenvalues and eigenvectors without explicitly working through B? Use the definitions of eigenvectors and eigenvalues.

Well I could say that

Det(B-\lamda I)=0 and put B=A+3I, then say that I can reduce A to a triangular matrix such that the diagonal elements(Eigenvalues of A)+(3-\lambda)=0 and solve from there. And get the e.values to be 7,4,5. But how would the e.vectors be altered?
 
Last edited:
rock.freak667 said:
Well I could say that

Det(B-\lamda I)=0 and put B=A+3I, then say that I can reduce A to a triangular matrix such that the diagonal elements(Eigenvalues of A)+(3+\lambda)=0 and solve from there. And get the e.values to be 7,4,5. But how would the e.vectors be altered?

The determinant is just a way to extract the eigenvalues mechanically. The original definition should state that L is an eigenvalue of A iff Av = Lv for some vector v (an eigenvector of A), or equivalently, A - LI = 0. Thus, to find eigenvalues L, we are looking for solutions of the equation B - LI = 0, which is equivalent to A + (3 - L)I = 0. We already know the solutions of this equation, specifically we know that L - 3 = L' where L' is an eigenvalue of A. No messing around with matrix forms or determinants necessary. Similarly, you can use the original equation to see how the eigenvectors of A compare to the eigenvectors of B. Can you see the way from here? :)
 
Last edited:
I got the same answer but it all amounts to the same thing I guess.thanks

But then the e.vectors would be unchanged?
 
rock.freak667 said:
I got the same answer but it all amounts to the same thing I guess.thanks

But then the e.vectors would be unchanged?

Indeed, you get the equation Av = (L - 3)v, where L is an eigenvalue of B and v is an eigenvector of B, implying that whenever v is an eigenvector for A, it is also an eigenvector for B, albeit with a different eigenvalue.
 
Last edited:
Question: A clock's minute hand has length 4 and its hour hand has length 3. What is the distance between the tips at the moment when it is increasing most rapidly?(Putnam Exam Question) Answer: Making assumption that both the hands moves at constant angular velocities, the answer is ## \sqrt{7} .## But don't you think this assumption is somewhat doubtful and wrong?

Similar threads

  • · Replies 3 ·
Replies
3
Views
2K
  • · Replies 2 ·
Replies
2
Views
1K
  • · Replies 7 ·
Replies
7
Views
2K
Replies
6
Views
1K
  • · Replies 8 ·
Replies
8
Views
2K
Replies
9
Views
2K
  • · Replies 12 ·
Replies
12
Views
3K
Replies
0
Views
848
Replies
2
Views
2K
  • · Replies 9 ·
Replies
9
Views
2K